LSAT and Law School Admissions Forum

Get expert LSAT preparation and law school admissions advice from PowerScore Test Preparation.

 moshei24
  • Posts: 465
  • Joined: Mar 20, 2012
|
#5658
I think the main reason this question gave me trouble was the words used in the stimulus, mainly at the end - "typically bland and innocuous."

The assumption in this question is that talks shows that will appeal to large numbers of people are the ones in which the political opinions and analyses are typically or almost always bland and innocuous. That would imply that only sometimes, if ever, will they air politics that are not bland or innocuous. But if there were politics that are not bland or innocuous that would draw large crowds, they would air them. That's why (B) is correct? Because if you negate that, it becomes "there are NOT television viewers who would refuse to watch television talk shows that they knew would be controversial and disturbing." And if that were so, then it would mean that EVERYONE would watch those shows, and that would mean that the fact that only sometimes they air those types of shows wouldn't be true - they would have to typically air those shows?

So the difference is a difference between some and most? Without that assumption, we would have to assume that most of the shows they air are controversial and disturbing, which wouldn't work with the fact that the they typically air the opposite? They can't typically air both?

By the way - controversial is the opposite of bland and disturbing is the opposite of innocuous, right? At least in this context.

This double trouble example is interesting because both questions are asking for an assumption. Never seen that before.

If someone could clear up my understanding of #4, I'd greatly appreciate it. Did I understand it properly? Did I overcomplicate it for myself?

Thanks for the help!

-Moshe
User avatar
 Dave Killoran
PowerScore Staff
  • PowerScore Staff
  • Posts: 5978
  • Joined: Mar 25, 2011
|
#5751
You more or less have it, I'd say. As you noted, the key is the last four lines of the stimulus, where they connect the premise about airing only shows that appeal to large numbers of people to the conclusion about the political opinions being aired are typically bland and innocuous. In making that connection, they have to believe that the bland shows appeal to large numbers, and that non-bland shows don't appeal. That's what (B) is driving at, that there have to be people out there who wouldn't watch non-bland shows.

Did you overcomplicate it? Maybe a bit, but that mostly occurred once you began negating answer choice (B). Aside from that, I think you had a clear understanding of the argument.

As you also note, controversial and disturbing is the opposite of bland and innocuous, and I'd probably say that it isn't just an opposite, but the polar opposite (this actually makes the answer easier to identify; it would have been the same if they said non-bland).

Finally, you are correct in referencing that double Assumption questions are a rare appearance, very rare indeed.

Thanks!
 moshei24
  • Posts: 465
  • Joined: Mar 20, 2012
|
#5760
Thank you!
 joshingschneider
  • Posts: 3
  • Joined: Sep 13, 2012
|
#6263
Hi,

Came across a question is PT 29 section 4 question 4. The Q stem states" An assumption made in the explanation offered by the author of the passage is that" to be honest was confused which question type this was relating to.
User avatar
 Dave Killoran
PowerScore Staff
  • PowerScore Staff
  • Posts: 5978
  • Joined: Mar 25, 2011
|
#6264
Hi JS,

That's just an odd way of asking a regular Assumption question. If you re-order the wording, it makes it a bit clearer:

  • "In the author's explanation, an assumption made is that"


Please let me know if that helps. Thanks!
 joshingschneider
  • Posts: 3
  • Joined: Sep 13, 2012
|
#6265
Thanks Dave, clarified the matter for me. Interesting to note, two assumption questions regarding the same stimulous. For question 5 on the same stimulous "The explanation offered by the author of the passage makes the assumption that" similar to the reordering you made of question 4.
User avatar
 Dave Killoran
PowerScore Staff
  • PowerScore Staff
  • Posts: 5978
  • Joined: Mar 25, 2011
|
#6266
Great, glad I could help! Yes, it's unusual for a double question to both be Assumptions, but it has happened at least one other time I can recall.

Thanks!
 kristinaroz93
  • Posts: 160
  • Joined: Jul 09, 2015
|
#20763
Hi,

I am still stumped with choice B of number 4. Anyone care to elaborate further, please? Are we using conditional logic for this one? Is this an assumption defender/supporter?

I fully understand 5, however, as it is a supporter assumption meant to fill in the gap between what is mainstream and what is typically bland and innocuous, as the author assumes these two to be one of the same.

Here is what I sort of think for 4: According to the stimulus, the reasons driving mainstream/bland shows being put on the station is purely economic. The argument says this is what appeals to most people, and so the more people you have watching a station the better that station is off economically. But whose to say people won't watch shows that aren't mainstream. What if just as many people would watch the station even if it had non mainstream shows (i.e. that are more controversial and disturbing)? Then that means economic reasons are not your main reason any longer for playing bland shows and the whole argument breaks down. So to keep the argument afloat, we need to
assume that people will stop watching the station the moment it airs non-mainstream shows. Let me know how i did and of course feel free to add on as much as possible to make it even clearer=)


(It is also strange to me that there are two assumption questions back to back in regards to the same stimulus!)
 Ricky_Hutchens
PowerScore Staff
  • PowerScore Staff
  • Posts: 59
  • Joined: Oct 12, 2015
|
#20828
Hi Kristin,

I think you've pretty much nailed the reasoning for why B is a necessary assumption for the argument. If large numbers of people of were willing to watch shows they found to be controversial and/or disturbing, then the entire argument would fall apart. This would mean there would have to be another reason these shows are not aired. You got it. Good job.

Ricky Hutchens
 kristinaroz93
  • Posts: 160
  • Joined: Jul 09, 2015
|
#21483
for number 4, though I understand why B is correct now, can someone please go over the impact of A and C on the argument. Or just these choices in general and how to interpret them when compared to the argument. I like to know that I have a good understanding of all other answer choices as well and why they are bad, what they are trying to say, and what bearing they have on the stimulus and its argument.

For instance, would A hurt the reasoning, because then it is not even possible to decide what mainstream even is if there is no consensus amongst the public on what they all agree is bland and would thus watch?

And with c, I am just completely stumped.

Thanks!

Get the most out of your LSAT Prep Plus subscription.

Analyze and track your performance with our Testing and Analytics Package.